What is 50% of 100%?

Answers

Answer 1
50/100
25/50 = 0.5
Logic
Answer 2

Answer:

We need to determine 50% of 100 now and the procedure explaining it as such

Step 1: In the given case Output Value is 100.

Step 2: Let us consider the unknown value as x.

Step 3: Consider the output value of 100 = 100%.

Step 4: In the Same way, x = 50%.

Step 5: On dividing the pair of simple equations we got the equation as under

100 = 100% (1).

x = 50% (2).

(100%)/(x%) = 100/50

Step 6: Reciprocal of both the sides results in the following equation

x%/100% = 50/100

Step 7: Simplifying the above obtained equation further will tell what is 50% of 100

x = 50%

Therefore, 50% of 100 is 50

Add to brainliest


Related Questions

Find the roots and the vertex of the quadratic on a calculator. Round all values to 3
decimal places (if necessary).
y=-x² + 6x + 72

Answers

Answers:
Zeros: x = (6,0), (-12,0)
Vertex: (-3, 45)

I attached a picture of the answer for the question. Hope this help!

Please help me I’m miserable because I was sick.It’s 2:00AM and this is due at 8 somebody please help me

Answers

Answer:

see explanation

Step-by-step explanation:

(a)

the 2 angles are same- side interior angles and are supplementary.

(c)

since they are supplementary , they sum to 180° , that is

6x + 14 + 10x - 10 = 180

16x + 4 = 180 ( subtract 4 from both sides )

16x = 176 ( divide both sides by 16 )

x = 11

(d)

then

6x + 14 = 6(11) + 14 = 66 + 14 = 80

10x - 10 = 10(11) - 10 = 110 - 10 = 100

note 80 + 100 = 180 ( supplementary angles )

if f(x) = 5x-7, then what is the solution to f(x) =4

Answers

The input value x when the output value f( x ) = 4 in the function f( x ) = 5x - 7 is 11/5.

What is the value of x when the output value f(x) = 4?

A function is simply a relationship that maps one input to one output, each x-value can only have one y-value.

Given the data in the question;

f( x ) = 5x - 7f(x) = 4x = ?

To determine the input value x when the output value f( x ) = 4, replace f( x ) by 4 in the function and solve for x.

f( x ) = 5x - 7

4 = 5x - 7

Collect like terms

4 + 7 = 5x

5x = 11

Divide both sides by 5

5x/5 = 11/5

x = 11/5

Therefore, the input value x is 11/5, this forms an ordered pair of ( 11/5, 4 ).

Lear more about functions here: brainly.com/question/2541698

#SPJ1

Kaj needs to order some new supplies for the restaurant where she works. The restaurant needs at least 736 knives. There are currently 155 knives. If each set on sale contains 12 knives, what is the minimum number of sets of knives Kaj should buy?

Answers

By using division, when the restaurant needs at least 736 knives and there are currently 155 knives, if each set on sale contains 12 knives then he minimum number of sets of knives Kaj should buy is 49 sets

The total number of knife that restaurant needs = 736 knives

Number of knives that restaurant has = 155 knives

The number of knife that he want to buy = 736-155

= 581

Number of knives on one set = 12

To find the number of set he wants to buy, we have to use division

Number of sets he wants to buy = 581/12

= 48.41

≈49 sets

Hence, by using division, when the restaurant needs at least 736 knives and there are currently 155 knives, if each set on sale contains 12 knives then he minimum number of sets of knives Kaj should buy is 49 sets

Learn more about division here

brainly.com/question/1575906

#SPJ1

At your child's birth, you begin contributing monthly to a college fund. The fund pays an APR of 4.1% compounded monthly. You figure your child will need $30,000 at age 18 to begin college. What monthly deposit is required?

Answers

the monthly deposit that will be required will be equal to $ 1214 by compound interest.

We are given the information that:

Amount (A) = $ 30,000

Time = 18 years

Compound interest rate = 4.1 %

Now, let the principal amount be x.

So, by using the formula for compound interest, we get that:

30,000 = x ( 1+ 0.041)¹⁸

30,000 = x (1.041)¹⁸

30,000 = x × 2.06

x = 30,000 / 2.06

x = 14563 ( approx.)

Monthly deposit = 14563 / 12 = 1214 (approx.)

Therefore, the monthly deposit that will be required will be equal to $ 1214 by compound interest.

Learn more about compound interest here:

https://brainly.com/question/24924853

#SPJ1

The graph of a function f(x) shown in orange is moved two units to the left to obtain the function in blue. Which of the following represents the blue graph

Answers

The equation of the blue line is y = 2(2 - x).

Equation of A(1,0) and B(0, -2)

Use y = mx + c,

m = change of x axis ÷ change of y axis

The difference in x-coordinates equals the difference in y-coordinates, and the difference in y-coordinates equals the difference in y-coordinates.

m = (y2 - y1) ÷ (x2 - x1)

To get the slope, plug in the parameters of both x and y into to the equation.

m = (2 - 0) ÷ (0 - 1)

m = -2

The value of b calculated using the expression of a line formula.

b = 2

Now that you have the measurements of m (slope) and b (y-intercept), plug them into y = m x + b to determine the equation of the line.

y = −2x+2

Because the line is two primary components to the left, the parallel line's equation is,

y = -2x + 2 + 2

y = -2x + 4

y = 2(2 - x)

Read more about line expressions at

https://brainly.com/question/21000669?referrer=searchResults

#SPJ1

Give the equation of the circle centered at the origin and passing through the point (4, 0).

Answers

[tex]\quad \huge \quad \quad \boxed{ \tt \:Answer }[/tex]

[tex]\qquad \tt \rightarrow \: x² + y² = 16[/tex]

____________________________________

[tex] \large \tt Solution \: : [/tex]

The distance between the centre and the point through which the circle is passing is equal to the radius of the circle.

so, let's use distance formula here :

[tex]\qquad \tt \rightarrow \: \sqrt{(y2 - y1) {}^{2} + (x2 - x1) {}^{2} } [/tex]

[tex]\qquad \tt \rightarrow \: \sqrt{(0 - 0) {}^{2} + (4 - 0) {}^{2} } [/tex]

[tex]\qquad \tt \rightarrow \: \sqrt{0 + (4) {}^{2} } [/tex]

[tex]\qquad \tt \rightarrow \: \sqrt{ {4}^{2} } [/tex]

[tex]\qquad \tt \rightarrow \: 4 \: \: units[/tex]

Now, let's write the equation of circle in standard form :

[tex]\qquad \tt \rightarrow \: (x - h) {}^{2} + (y - k) {}^{2} = r {}^{2} [/tex]

h = x - coordinate of circle k = y - coordinate of circle r = radius of circle

[tex]\qquad \tt \rightarrow \: (x - 0) {}^{2} + (y - 0) {}^{2} = {4}^{2} [/tex]

[tex]\qquad \tt \rightarrow \: {x}^{2} + {y}^{2} = 16 [/tex]

Answered by : ❝ AǫᴜᴀWɪᴢ ❞

Does anybody know how to answer this to?

The table shows the relationship between the participants walking and running for the week's cross-country practices.


Walk (laps) 3 B 15
Run (laps) 5 10 D
Total (laps) A C 40


At this rate, how many laps will the participants walk if the total distance is 32 miles? How many miles will they run?

They will walk 7 laps and run 17 laps for a total of 32 miles.
They will walk 12 laps and run 20 laps for a total of 32 miles.
They will walk 14 laps and run 18 laps for a total of 32 miles.
They will walk 10 laps and run 22 laps for a total of 32 miles.

Question 3(Multiple Choice Worth 2 points)
(Three-Column Tables LC)

PART 2

The data below shows the ratio of brown to green crayons in four kindergarten classrooms.

12 to 20
10 to 25
14 to 35
30 to 75

Which table below correctly shows these ratios in a three-column table?


Brown 20 25 35 75
Green 12 10 14 30
Total 32 35 49 105

Brown 12 10 14 75
Green 20 25 35 30
Total 30 35 44 105

Brown 12 10 14 30
Green 20 25 35 75
Total 32 35 49 105

Brown 12 25 14 30
Green 20 10 35 75
Total 42 35 49 100

Answers

Using proportional relationships, it is found that:

They will walk 12 laps and run 20 laps for a total of 32 miles.The correct option for the second problem is:

Brown 12 10 14 30

Green 20 25 35 75

Total 32 35 49 105

What is a direct proportional relationship?

In a direct proportional relationship, the output variable is found by the multiplication of the input variable and the constant of proportionality k, as follows:

y = kx.

For the first question, we have that out of a total distance of 8 miles, they:

Walk 3/8 of the distance.Run 5/8 of the distance.

Hence the proportional relationships for the distances are given as follows:

Walked = 3/8 x Total Distance.Ran = 5/8 x Total Distance.

For a total distance of 32 miles, the distances walked and ran are given as follows:

Walked: 3/8 x 32 = 3 x 4 = 12 miles = 12 laps.Ran: 5/8 x 32 = 5 x 4 = 20 miles = 20 laps.

For the second question, we have that the table has to following the given ratios of the first row to the second row, with the third row being the sum of the first two rows, hence the third table is correct.

More can be learned about proportional relationships at https://brainly.com/question/10424180

#SPJ1

Connor edits 34
pages in 8 hours. Complete the table using equivalent ratios

Answers

If Connor edits 34 pages in 8 hours. Using the equivalent ratios, the table has been completed.

The equivalent ratios are the ratios that has same value when we compare them. We can check two or more ratios and check whether they are equivalent or not.

Connor edits 34 pages in 8 hours

Number of pages he edits in 8 hours = 34 pages

Number of pages he edits in 1 hour = 34/8 = 17/4 pages

Number of pages he edits in 12 hours = (17/4)×12

= 51 pages

Complete the table

Hence, if Connor edits 34 pages in 8 hours, using the equivalent ratios, the table has been completed

Learn more about equivalent ratio here

brainly.com/question/13513438

#SPJ1

pls help me with this

Answers

When solving an inequality, treat it like an equal sign while solving.

y - 4 ≤ -13
y - 4 + 4 ≤ -13 + 4
y ≤ -9

Can you please help me with this

Answers

Let's classify the angles.

3.

The angles are alternate interior angles.

4.

The

David requires at least $250 to hold his birthday party. If David can save $43 a month, how many months will he need to save to be able to afford his birthday party?

a. Write an inequality that describes this situation.

b. Solve the inequality. Show all your work.

c. Write your answer in a complete sentence.

Answers

Answer:

Step-by-step explanation:

Let David will need m months to save to able to afford his birthday party.

Then 43m>=250 that is m>=5.814 .

But since months are whole numbers

So We take m>=6.

He will need at least 6 months to save to able to afford his birthday party.

The answer is 6 months .

A motorboat travels 282 kilometers in 6 hours going upstream. It travels 402 kilometers going downstream in the same amount of time. What is the rate of the boat in still water and what is the rate of the current?

Answers

Let,

B = the speed of the boat in still water

S = the speed of the stream

Use the equation for distance.

Rate * Time = Distance

Upstream:

(B-S)6 = 282

Downstream:

(B+S)6 = 402

Simply first:

(B-S)6 = 282

(B-S) = 282/6

B-S=47

(B+S)6 = 402

(B+S) = 402/6

B+S=67

Solve by elimination. Add the two equations.

B-S=47

+ B+S=67

2B = 114

B = 114/2

B = 57 km/hr (the speed of the boat in still water)

Substitute this into either equation and solve for S.

B+S=67

57 + S = 67

S = 67 - 57

S = 10 km/hr (the speed of the stream)

11. Does the table show a proportional relationship?
If so, what is the value of y when x is 10?


X 5 6 7 8

Y 1 2/3 2 2 1/3 2 2/3

Answers

The value of y when x is 10 is 3[tex]\frac{1}{3}[/tex].

What is proportional relationship?

Relationships between two variables that are proportional occur when their ratios are equal. Another way to consider them is that in a proportionate relationship, one variable is consistently equal to the other's constant value. The "constant of proportionality" is the name of this constant. The equation y = kx represents a proportionate relationship between two quantities y and x that have the same proportionality constant, k. Any relationship that has a constant ratio is said to be proportionate. For instance, the ratio of proportionality is the average number of apples per tree, and the amount of apples in a crop is proportional to the number of trees in the orchard.

Given Data

X 5 6 7 8

Y 1 2/3 2 2 1/3 2 2/3

Yes, the table shows proportional relationship.

Value of Y when X will be 10 is

x = 10

= 3[tex]\frac{1}{3}[/tex]

the value of y when x is 10 is 3[tex]\frac{1}{3}[/tex].

To learn more about proportional relationship, visit:

https://brainly.com/question/12917806

#SPJ9

2 + 31 x 1/10 + 4 x 1/1000 is greater than, less than, or equal to 2.324?

Answers

Answer:

greater

Step-by-step explanation:

if it wasn't bidmas then the product would be 5.104 which is greater than 2.324

Write the following sentence in algebraic language:Number k is 4/5 of the number t.

Answers

To write the sentence:

Number k is 4/5 of the number t in algebraic language, we can proceed as follows:

[tex]k=\frac{4}{5}t[/tex]

We need to express that k is equal to the four-fifths of t.

Then, the answer is k = (4/5)*t.

If you answer these questions correctly you are really smart. What is 2+2*2, what is 3+3*3, what is 6+6*6.

Answers

The value of the numbers that are given will be:

1. 6

2. 12

3. 42

How to calculate the value?

It should be noted that based on the information given, PEDMAS is important. This implies that multiplication will come first before addition. This will be illustrated thus:

1. 2 + 2 × 2

= 2 + 4

= 6.

2. 3 + 3 × 3

= 3 + 9

= 12

3. 6 + 6 × 6

= 6 + 36

= 42..

Therefore, it should be noted that we multiplied first before adding.

Learn more about numbers on:

brainly.com/question/11418015

#SPJ1

Find the inequality that creates the following graph

Answers

Answer:

|2x - 3| ≥ 1

Step-by-step explanation:

Since the absolute value function is always ≥ 0 we can conclude that                
|2x - 3| ≥ 0 so we can eliminate choices 2 and 4

That leaves two possibilities:
|2x - 3| ≥ 1  and |2x - 3| ≤  1

There are two ways to go about solving this

Pick a point in the given graph on each side of the segment marked in red and see which of the two inequalities both points satisfy

Two points which are on both segments are x = 0 and x = 1

Plug x = 0 into |2x - 3|
|2x - 3| at x = 0
=> | 2 ·0 - 3|  
=> |0 - 3|
=> |-3|
= 3  

Since 3 ≥ 1,  point x = 0 satisfies |2x - 3| ≥ 1

But it does not satisfies |2x - 3| ≤  1 since 3 is not less than or equal to 1

So the correct inequality is |2x - 3| ≥ 1

Indicate whether the following statement is
TRUE or FALSE. −17 is an integer

Answers

Answer:

TRUE

Step-by-step explanation:

an integer is a whole number from the set of negative, non-negative, positive and 0 numbers.

ex : -1, -2, -3, -4, -5, 0, 1, 2, 3, 4, 5

-17 is a negative whole number

so yes, -17 is an integer

I need help I don’t know how to do this

Answers

do do this we have to multiply the common factors
41.2y^2 (meaning y is squared)

In anatomy, a student learned that the average resting heart rate is between 60 and 100 beats per minute. The student decided to record the heart rate of people over five minutes while waiting in line at the pharmacy. The dot plot shows the results.

Dot plot with 1 dot at 62, 3 dots at 68, 1 dot at 69, 2 dots at 70, 3 dots at 72, 2 dots at 75, 1 dot at 76, 2 dots at 78, 3 dots at 80, and 2 dots at 89

Which statement below best describes the shape of the distribution?

The data is roughly symmetrical distributed, with most values clustered from 68 to 80 beats per minute. The values at 62 and 89 are possible outliers. The data fits within the average of 60 to 100 beats per minute.
The data is not symmetrically distributed, with most values clustered from 68 to 80 beats per minute. The values at 62 and 89 are possible outliers. The data fits within the average of 60 to 100 beats per minute.
The data is skewed right, with fewer values on the right end of the graph. The values at 62 and 89 are possible outliers. The data fits within the average of 60 to 100 beats per minute.
The data is skewed left, with fewer values on the left end of the graph. The values at 62 and 89 are possible outliers. The data fits within the average of 60 to 100 beats per minute.

Answers

The shape of the distribution for the given dot plot is as follows:

The data is skewed right, with fewer values on the right end of the graph. The values at 62 and 89 are possible outliers. The data fits within the average of 60 to 100 beats per minute.

What does a dot plot show?

A dot plot shows the number of times that each measure appears in the data-set.

In the context of this problem, we have that from the given dot plot, the measures are as follows:

62, 68, 68, 68, 69, 70, 70, 72, 72, 72, 75, 75, 76, 78, 78, 80, 80, 80, 89, 89.

The mean is the sum of all observations divided by the number of observations, hence, applying it's formula, it is given by:

74.55.

The median is the middle value of the data-set, the value of which 50% of the measures are greater and 50% are less. The data-set in this problem has 20 elements, hence the median is the mean of the 10th and the 11th elements, which are 72 and 75, respectively, hence:

Median = (72 + 75)/2 = 73.5.

The median is less than the median, hence the distribution is skewed to the right, which means that the third option is correct.

More can be learned about dot plots at https://brainly.com/question/25957672

#SPJ1

Answer:

The data is not symmetrically distributed, with most values clustered from 68 to 80 beats per minute. The values at 62 and 89 are possible outliers. The data fits within the average of 60 to 100 beats per minute.

Step-by-step explanation:

its not skewed right and this is the next one that makes sense

Norman buys a new car for $21,500. The simple interest rate is 4.2% and the amount of loan (plus simple interest) is repayable in 5 years. What is the total amount that must be repaid?

Round your answer to the nearest dollar and do not round until the final answer.

Answers

The total amount that must be repaid will be $26,015.

What is simple interest?

Simple interest is the concept that is used in many companies such as banking, finance, automobile, and so on.

A = P + (PRT)/100

Where P is the principal, R is the rate of interest, and T is the time.

For $21,500, Norman buys a brand-new vehicle. The loan amount (plus simple interest) must be repaid in full within five years, and the simple interest rate is 4.2%.

The total amount that must be repaid will be given as,

A = $21,500 + ($21,500 x 4.2 x 5)/100

A = $21,500 + $4,515

A = $26,015

The total amount that must be repaid will be $26,015.

More about the simple interest link is given below.

https://brainly.com/question/2793278

#SPJ1

Find the value of 2y-9 given that -3y-2=4.
Simplify your answer as much as possible.
2y - 9 =

Answers

In linear equation, 4 is the Simplify your answer .

What is a linear equation example?

Ax+By=C is the usual form for two-variable linear equations.As an illustration, the conventional form of the linear equation 2x+3y=5 When an equation is given in this format, finding both intercepts is rather simple (x and y).A linear equation is a first-order (linear) term plus a constant in the algebraic form y=mx+b, where m is the slope and b is the y-intercept.The variables in the previous sentence, y and x, are referred to as a "linear equation with two variables" at times.

2y-9 ............1

-3y-2=4 .............2

From equation  (2)

                  - 3y = 4 + 2

                  - 3y = 6

                     y = 6/-3  ⇒ -2

value of y put in equation 2

                         -3y-2=4

                       - 3 * -2 - 2 = 4 ⇒ 6 - 2 ⇒ 4

Learn more about linear equation

brainly.com/question/11897796

#SPJ13

                     

1) Find the measures of the following angles: a) 2x+250 3x-450 t b) e, I l l 12y by-30° g

Answers

a) the two angles are supplementary, then:

(3x - 45) + (2x + 25) = 180

(3x + 2x) + (-45 + 25) = 180

5x - 20 = 180

5x = 180 + 20

5x = 200

x = 200/5

x = 40

Then, the measure of the angles are:

3x - 45 = 3*40 - 45 = 75°

2x + 25 = 2*40 + 25 = 105°

b) the two angles are complementary, then:

(2y) + (6y - 30) = 90

(2y + 6y) - 30 = 90

8y = 90 + 30

8y = 120

y = 120/8

y = 15

Then, the measure of the angles are:

2y = 2*15 = 30°

6y - 30 = 6*15 - 30 = 60°

c) the two angles are vertical angles, then they are congruent, that is,

2z + 7 = 5z - 23

7 + 23 = 5z - 2z

30 = 3z

30/3 = z

10 = z

Then, the measure of the angles are:

2z + 7 = 2*10 + 7 = 27°

5z - 23 = 5*10 - 23 = 27°

d) the two angles are alternative exterior angles, then they are congruent, that is,

4x - 5 = 3x + 15

4x - 3x = 15 + 5

x = 20

Then, the measure of the angles are:

4x - 5 = 4*20 - 5 = 75°

3x + 15 = 3*20 + 15 = 75°

3 times the sum of a number and 5

SHOW WORK

Answers

Your answer will be 3x+5

Answer:

  3(n +5)

Step-by-step explanation:

You want the math expression that means "three times the sum of a number and five."

Sum

A sum is represented using a plus sign. "A number" is represented using a variable. "x" is a variable commonly used for an unknown value. "n" can also be used to represent "a number."

The sum of a number and 5 will be ...

  n + 5

Times

We want an expression that is 3 times that sum, so we multiply it by 3:

  3(n +5) . . . . . . 3 times the sum of a number and 5

__

Additional comment

Note that the expression above is not the same as 3n+5, which would represent "the sum of 3 times a number and 5".

write and solve the inequality that represents -1/3 is > than or = to the product of -4/5 and a number

Answers

Answer:

[tex] - \frac{1}{3} \geqslant - \frac{4}{5} x[/tex]

[tex] \frac{1}{3} \leqslant \frac{4}{5} x[/tex]

[tex]5 \leqslant 12x[/tex]

[tex]12x \geqslant 5[/tex]

[tex]x \geqslant \frac{5}{12} [/tex]

What is the answer to this question

Answers

Given that tanθ = 3.2603 where π < θ < 3π/2

[tex]\begin{gathered} \text{sin}\theta\approx0.9560 \\ \cos \theta\approx0.2932 \end{gathered}[/tex][tex]\begin{gathered} \sin ^2\theta+\cos ^2\theta=1 \\ \text{where} \\ \tan \theta=\frac{\sin \theta}{\cos \theta} \end{gathered}[/tex]

From the calculation the value of θ = 72.95, hence the correct the error by adding 180

because it dint dnt fall in the range of

[tex]\pi<\theta<\frac{3\pi}{2}[/tex]

And to correct the error add 180 to their answer,

Hence the value of θ = 180 + 72.95 = 252.95 degree

The prices of the 19 top-rated all-season fires for a specific tire size, are as follows. Answer parts (a)-(0)$87 $116 $98 579 582 592 594 589 594 $81$107 $112 5103 596 586 $92 $75 $99 $90a) Determine Q2b) Determine Q1c) Determine Q3

Answers

Simone, these are the steps to calculate Q1, Q2, and Q3.

Step 1: You need to put all the values in order from the lowest to the greatest.

Step 2: The tenth value from left to right is the median or Q2.

Step 3: The

7+4(1-8p)
I’m confused is it -32p+11 or 11-32p

Answers

Answer:

Either. They are both correct

Step-by-step explanation:

So 7+4(1-8p): 7+4-32p => 11-32p => -32p+11.

Answer:

11-32p

Step-by-step explanation:

The answer is 11-32p because your only distributing the 4 to the 1 and 8p.

hello, the question is in the picture :)

Answers

By cross multiplying the given equation, we have

[tex]6x=(x-2)(x+8)[/tex]

By multiplying the right hand side, we get

[tex]6x=x^2+6x-16[/tex]

Then, by subtracting 6x to both sides, we have

[tex]0=x^2-16[/tex]

or equivalently,

[tex]x^2-16=0[/tex]

Since the left hand side is a conjugate binomial, it can be expressed as

[tex]x^2-16=(x+4)(x-4)[/tex]

Then, we have the equation

[tex](x+4)(x-4)=0[/tex]

Therefore, the values of x that make the equation true are x= 4 and x= -4

Other Questions
Can someone do questions, 1, 2 and 3 if possible please? f(x)=x-7; vertical stretch of 3 g(x) What was John proctors actual relationship with Giles Corey? Use the following information to find x: AB = 4x + 5; BC = 13; and B is the midpoint of AC A runner starting from rest reaches a velocity of 9.6 m/s in 2.0s. What is her average acceleration? Consider the following information: Cash = $6,000 Beginning Balance = $200,000 Capital added = $15,000 Revenues = $10,000 Expenses = $5,000 Net Income = $56,000 Owners Withdrawals = $60,000 What is the ending balance on the Statement of Changes in Owner's Equity for this data? . TEST SCORES Benjamin's test scores for the quarter so far are 78, 82, 95, and 88. What does he need to score on the fifth test of the quarter in order to have an average test score of 87? 9(4)^2 + 24(4) + 20 = 4 HELPPPPPPPPPPPPPPPPP Read this sentence: The euphoria Kaya felt when she learned that she got accepted into UCF was expressed by joyful screaming and dancing around her room. Which word from the sentence provides a clue to the meaning of euphoria? Accepted Joyful Learned Screaming answer the question on the image for 20 points what is the value of x-y if y=4 and 3/5 / 1/2 = x/10like 3/5___1/2F.) 4G.) 8H.) 10J.) 12K.) 16pls show work Please solve this question!! Please help me Find the y-component of thisvector:22.3 m77.1Remember, angles are measured fromthe +x axis. ______ are formed in stars cores as a result of fusion.Ultraviolet wavesRadio wavesGamma raysInfrared waves your patient presents with symptoms that lead you to suspect acute appendicitis. which assessment finding is least likely to be associated with this condition early in its course? 7,5,2,1,.......tell the common difference of this sequence Define the term Saccharomyces Ty :)) The word that has the/ j/ sound Describe what Hippocrates was famous for. 4(NO3)2please explain, will give brainliest!!